zero times infinity in an integral











up vote
0
down vote

favorite












Suppose $f$ and $g$ are measurable functions defined on a ball $B$ of $mathbb{R}^n$ ($ngeq1$), satisfying
$$0leqint_{B}f(x)g(x)dx=f(x_{0})int_{B}g(x)dx,$$
for some $x_{0}in B$. My question is: if $f(x_{0})=0$, can we say that $int_{B}g(x)dx$ is not $infty$?










share|cite|improve this question


























    up vote
    0
    down vote

    favorite












    Suppose $f$ and $g$ are measurable functions defined on a ball $B$ of $mathbb{R}^n$ ($ngeq1$), satisfying
    $$0leqint_{B}f(x)g(x)dx=f(x_{0})int_{B}g(x)dx,$$
    for some $x_{0}in B$. My question is: if $f(x_{0})=0$, can we say that $int_{B}g(x)dx$ is not $infty$?










    share|cite|improve this question
























      up vote
      0
      down vote

      favorite









      up vote
      0
      down vote

      favorite











      Suppose $f$ and $g$ are measurable functions defined on a ball $B$ of $mathbb{R}^n$ ($ngeq1$), satisfying
      $$0leqint_{B}f(x)g(x)dx=f(x_{0})int_{B}g(x)dx,$$
      for some $x_{0}in B$. My question is: if $f(x_{0})=0$, can we say that $int_{B}g(x)dx$ is not $infty$?










      share|cite|improve this question













      Suppose $f$ and $g$ are measurable functions defined on a ball $B$ of $mathbb{R}^n$ ($ngeq1$), satisfying
      $$0leqint_{B}f(x)g(x)dx=f(x_{0})int_{B}g(x)dx,$$
      for some $x_{0}in B$. My question is: if $f(x_{0})=0$, can we say that $int_{B}g(x)dx$ is not $infty$?







      real-analysis integration






      share|cite|improve this question













      share|cite|improve this question











      share|cite|improve this question




      share|cite|improve this question










      asked Nov 29 at 5:26









      M. Rahmat

      309211




      309211






















          2 Answers
          2






          active

          oldest

          votes

















          up vote
          2
          down vote



          accepted










          You cannot say that. Take $fequiv 0$ and a $g$ whose integral is $infty$.






          share|cite|improve this answer




























            up vote
            1
            down vote













            This still works if $int_Bg = infty$.



            The Lebesgue integral is defined for extended real-valued functions and the definition $0 cdot infty = 0$ is imposed. This is necessary to insure consistency.



            The integral of a nonnegative function over a measurable E is defined as



            $$int_E f = sup left{ int_E g ,,| ,,g,, text{measurable, of compact support, and } , 0 leqslant g leqslant fright} $$
            and if $f = 0$ we have $int_E f = 0$ even if $m(E) = infty$



            Hence,



            $$0 = int_E 0 = 0 cdot m(E) = 0 cdot infty$$



            Thus, if $f(x_0) = 0$ and $int_Bg = infty$ then



            $$f(x_0) int_Bg = 0$$






            share|cite|improve this answer





















              Your Answer





              StackExchange.ifUsing("editor", function () {
              return StackExchange.using("mathjaxEditing", function () {
              StackExchange.MarkdownEditor.creationCallbacks.add(function (editor, postfix) {
              StackExchange.mathjaxEditing.prepareWmdForMathJax(editor, postfix, [["$", "$"], ["\\(","\\)"]]);
              });
              });
              }, "mathjax-editing");

              StackExchange.ready(function() {
              var channelOptions = {
              tags: "".split(" "),
              id: "69"
              };
              initTagRenderer("".split(" "), "".split(" "), channelOptions);

              StackExchange.using("externalEditor", function() {
              // Have to fire editor after snippets, if snippets enabled
              if (StackExchange.settings.snippets.snippetsEnabled) {
              StackExchange.using("snippets", function() {
              createEditor();
              });
              }
              else {
              createEditor();
              }
              });

              function createEditor() {
              StackExchange.prepareEditor({
              heartbeatType: 'answer',
              autoActivateHeartbeat: false,
              convertImagesToLinks: true,
              noModals: true,
              showLowRepImageUploadWarning: true,
              reputationToPostImages: 10,
              bindNavPrevention: true,
              postfix: "",
              imageUploader: {
              brandingHtml: "Powered by u003ca class="icon-imgur-white" href="https://imgur.com/"u003eu003c/au003e",
              contentPolicyHtml: "User contributions licensed under u003ca href="https://creativecommons.org/licenses/by-sa/3.0/"u003ecc by-sa 3.0 with attribution requiredu003c/au003e u003ca href="https://stackoverflow.com/legal/content-policy"u003e(content policy)u003c/au003e",
              allowUrls: true
              },
              noCode: true, onDemand: true,
              discardSelector: ".discard-answer"
              ,immediatelyShowMarkdownHelp:true
              });


              }
              });














              draft saved

              draft discarded


















              StackExchange.ready(
              function () {
              StackExchange.openid.initPostLogin('.new-post-login', 'https%3a%2f%2fmath.stackexchange.com%2fquestions%2f3018224%2fzero-times-infinity-in-an-integral%23new-answer', 'question_page');
              }
              );

              Post as a guest















              Required, but never shown

























              2 Answers
              2






              active

              oldest

              votes








              2 Answers
              2






              active

              oldest

              votes









              active

              oldest

              votes






              active

              oldest

              votes








              up vote
              2
              down vote



              accepted










              You cannot say that. Take $fequiv 0$ and a $g$ whose integral is $infty$.






              share|cite|improve this answer

























                up vote
                2
                down vote



                accepted










                You cannot say that. Take $fequiv 0$ and a $g$ whose integral is $infty$.






                share|cite|improve this answer























                  up vote
                  2
                  down vote



                  accepted







                  up vote
                  2
                  down vote



                  accepted






                  You cannot say that. Take $fequiv 0$ and a $g$ whose integral is $infty$.






                  share|cite|improve this answer












                  You cannot say that. Take $fequiv 0$ and a $g$ whose integral is $infty$.







                  share|cite|improve this answer












                  share|cite|improve this answer



                  share|cite|improve this answer










                  answered Nov 29 at 5:42









                  Kavi Rama Murthy

                  48.2k31854




                  48.2k31854






















                      up vote
                      1
                      down vote













                      This still works if $int_Bg = infty$.



                      The Lebesgue integral is defined for extended real-valued functions and the definition $0 cdot infty = 0$ is imposed. This is necessary to insure consistency.



                      The integral of a nonnegative function over a measurable E is defined as



                      $$int_E f = sup left{ int_E g ,,| ,,g,, text{measurable, of compact support, and } , 0 leqslant g leqslant fright} $$
                      and if $f = 0$ we have $int_E f = 0$ even if $m(E) = infty$



                      Hence,



                      $$0 = int_E 0 = 0 cdot m(E) = 0 cdot infty$$



                      Thus, if $f(x_0) = 0$ and $int_Bg = infty$ then



                      $$f(x_0) int_Bg = 0$$






                      share|cite|improve this answer

























                        up vote
                        1
                        down vote













                        This still works if $int_Bg = infty$.



                        The Lebesgue integral is defined for extended real-valued functions and the definition $0 cdot infty = 0$ is imposed. This is necessary to insure consistency.



                        The integral of a nonnegative function over a measurable E is defined as



                        $$int_E f = sup left{ int_E g ,,| ,,g,, text{measurable, of compact support, and } , 0 leqslant g leqslant fright} $$
                        and if $f = 0$ we have $int_E f = 0$ even if $m(E) = infty$



                        Hence,



                        $$0 = int_E 0 = 0 cdot m(E) = 0 cdot infty$$



                        Thus, if $f(x_0) = 0$ and $int_Bg = infty$ then



                        $$f(x_0) int_Bg = 0$$






                        share|cite|improve this answer























                          up vote
                          1
                          down vote










                          up vote
                          1
                          down vote









                          This still works if $int_Bg = infty$.



                          The Lebesgue integral is defined for extended real-valued functions and the definition $0 cdot infty = 0$ is imposed. This is necessary to insure consistency.



                          The integral of a nonnegative function over a measurable E is defined as



                          $$int_E f = sup left{ int_E g ,,| ,,g,, text{measurable, of compact support, and } , 0 leqslant g leqslant fright} $$
                          and if $f = 0$ we have $int_E f = 0$ even if $m(E) = infty$



                          Hence,



                          $$0 = int_E 0 = 0 cdot m(E) = 0 cdot infty$$



                          Thus, if $f(x_0) = 0$ and $int_Bg = infty$ then



                          $$f(x_0) int_Bg = 0$$






                          share|cite|improve this answer












                          This still works if $int_Bg = infty$.



                          The Lebesgue integral is defined for extended real-valued functions and the definition $0 cdot infty = 0$ is imposed. This is necessary to insure consistency.



                          The integral of a nonnegative function over a measurable E is defined as



                          $$int_E f = sup left{ int_E g ,,| ,,g,, text{measurable, of compact support, and } , 0 leqslant g leqslant fright} $$
                          and if $f = 0$ we have $int_E f = 0$ even if $m(E) = infty$



                          Hence,



                          $$0 = int_E 0 = 0 cdot m(E) = 0 cdot infty$$



                          Thus, if $f(x_0) = 0$ and $int_Bg = infty$ then



                          $$f(x_0) int_Bg = 0$$







                          share|cite|improve this answer












                          share|cite|improve this answer



                          share|cite|improve this answer










                          answered Nov 29 at 6:13









                          RRL

                          48.5k42573




                          48.5k42573






























                              draft saved

                              draft discarded




















































                              Thanks for contributing an answer to Mathematics Stack Exchange!


                              • Please be sure to answer the question. Provide details and share your research!

                              But avoid



                              • Asking for help, clarification, or responding to other answers.

                              • Making statements based on opinion; back them up with references or personal experience.


                              Use MathJax to format equations. MathJax reference.


                              To learn more, see our tips on writing great answers.





                              Some of your past answers have not been well-received, and you're in danger of being blocked from answering.


                              Please pay close attention to the following guidance:


                              • Please be sure to answer the question. Provide details and share your research!

                              But avoid



                              • Asking for help, clarification, or responding to other answers.

                              • Making statements based on opinion; back them up with references or personal experience.


                              To learn more, see our tips on writing great answers.




                              draft saved


                              draft discarded














                              StackExchange.ready(
                              function () {
                              StackExchange.openid.initPostLogin('.new-post-login', 'https%3a%2f%2fmath.stackexchange.com%2fquestions%2f3018224%2fzero-times-infinity-in-an-integral%23new-answer', 'question_page');
                              }
                              );

                              Post as a guest















                              Required, but never shown





















































                              Required, but never shown














                              Required, but never shown












                              Required, but never shown







                              Required, but never shown

































                              Required, but never shown














                              Required, but never shown












                              Required, but never shown







                              Required, but never shown







                              Popular posts from this blog

                              Different font size/position of beamer's navigation symbols template's content depending on regular/plain...

                              Berounka

                              I want to find a topological embedding $f : X rightarrow Y$ and $g: Y rightarrow X$, yet $X$ is not...